Implicit function theorem proof question

Click For Summary
The discussion centers on confusion regarding the implicit function theorem, specifically the transition between equations 8.1-4 and 8.1-5, where it suggests considering F as a function of x and z only. The user seeks clarification on the plausibility of this approach in proving the implicit solution f(x,y) of F(x,y,z)=c. A request for clearer visual aids is made, as the provided image is unreadable. The focus remains on understanding the reasoning behind the selective consideration of variables in the proof. Clear communication of the relevant equations is necessary for further assistance.
Syrus
Messages
213
Reaction score
0

Homework Statement


I understand the proof of the implicit function theorem up to the point in which I have included a photo. This portion serves to prove the familiar equation for the implicit solution f(x,y) of F(x,y,z)=c. My confusion arises between equations 8.1-4 and 8.1-5 when it is stated to "consider F as a function of x and z only." Why is this at all plausible?

Homework Equations

The Attempt at a Solution

 

Attachments

  • image.png
    image.png
    29.7 KB · Views: 533
Physics news on Phys.org
Pardon me, I meant partial derivative fx of the implicit solution f(x,y) of F(x,y,z).
 
Syrus said:

Homework Statement


I understand the proof of the implicit function theorem up to the point in which I have included a photo. This portion serves to prove the familiar equation for the implicit solution f(x,y) of F(x,y,z)=c. My confusion arises between equations 8.1-4 and 8.1-5 when it is stated to "consider F as a function of x and z only." Why is this at all plausible?

Homework Equations

The Attempt at a Solution


Your image is much too small and is quite fuzzy---basically unreadable. Please type out the relevant line or two that contains your actual question.
 
Question: A clock's minute hand has length 4 and its hour hand has length 3. What is the distance between the tips at the moment when it is increasing most rapidly?(Putnam Exam Question) Answer: Making assumption that both the hands moves at constant angular velocities, the answer is ## \sqrt{7} .## But don't you think this assumption is somewhat doubtful and wrong?

Similar threads

  • · Replies 1 ·
Replies
1
Views
1K
  • · Replies 6 ·
Replies
6
Views
1K
Replies
10
Views
2K
Replies
8
Views
2K
  • · Replies 2 ·
Replies
2
Views
2K
Replies
5
Views
2K
  • · Replies 5 ·
Replies
5
Views
2K
  • · Replies 27 ·
Replies
27
Views
2K
  • · Replies 1 ·
Replies
1
Views
1K
  • · Replies 3 ·
Replies
3
Views
2K